ChaseDream
搜索
返回列表 发新帖
楼主: sdcar2010
打印 上一主题 下一主题

SDCAR2010【逻辑入门】(八)Assumptions

[精华] [复制链接]
11#
 楼主| 发表于 2011-6-26 21:16:53 | 只看该作者
Again, time to prick your brain! Enjoy. The answers and explanations will follow.

1. A recent report determined that although only 3 percent of drivers on Maryland highways equipped their vehicles with radar detectors, 33 percent of all vehicles ticketed for exceeding the speed limit were equipped with them. Clearly, drivers who equip their vehicles with radar detectors are more likely to exceed the speed limit regularly than are drivers who do not.
The conclusion drawn above depends on which of the following assumptions?
(A) Drivers who equip their vehicles with radar detectors are less likely to be ticketed for exceeding the speed limit than are drivers who do not.
(B) Drivers who are ticketed for exceeding the speed limit are more likely to exceed the speed limit regularly than are drivers who are not ticketed.
(C) The number of vehicles that were ticketed for exceeding the speed limit was greater than the number of vehicles that were equipped with radar detectors.
(D) Many of the vehicles that were ticketed for exceeding the speed limit were ticketed more than once in the time period covered by the report.
(E) Drivers on Maryland highways exceeded the speed limit more often than did drivers on other state highways not covered in the report.

2. A researcher discovered that people who have low levels of immune-system activity tend to score much lower on tests of mental health than do people with normal or high immune-system activity. The researcher concluded from this experiment that the immune system protects against mental illness as well as against physical disease.
The researcher's conclusion depends on which of the following assumptions?
(A) High immune-system activity protects against mental illness better than normal immune system activity does.
(B) Mental illness is similar to physical disease in its effects on body systems.
(C) People with high immune-system activity cannot develop mental illness.
(D) Mental illness does not cause people's immune system activity to decrease.
(E) Psychological treatment of mental illness is not as effective as is medical treatment.

3. In Kravonia, the average salary for jobs requiring a college degree has always been higher than the average salary for jobs that do not require a degree. Current enrollments in Kravonia's colleges indicate that over the next four years the percentage of the Kravonian workforce with college degrees will increase dramatically.  Therefore, the average salary for all workers in Kravonia is likely to increase over the next four years.
Which of the following is an assumption on which the argument depends?
A. Kravonians with more than one college degree earn more, on average, than do Kravonians with only one college degree.
B. The percentage of Kravonians who attend college in order to earn higher salaries is higher now than it was several years ago.
C. The higher average salary for jobs requiring a college degree is not due largely to a scarcity among the Kravonian workforce of people with a college degree.
D. The average salary in Kravonia for jobs that do not require a college degree will not increase over the next four years.
E. Few members of the Kravonian workforce earned their degrees in other countries.

4. A significant number of complex repair jobs carried out by Ace Repairs have to be reworked under the company’s warranty. The reworked jobs are invariably satisfactory. When initial repairs are inadequate, therefore, it is not because the mechanics lack competence; rather, there is clearly a level of focused concentration that complex repairs require that is elicited more reliably by rework jobs than by first-time jobs.
The argument above assumes which of the following?
A. There is no systematic difference in membership between the group of mechanics who do first-time jobs and the group of those who do rework jobs.
B. There is no company that successfully competes with Ace Repairs for complex repair jobs.
C. Ace Repairs’ warranty is good on first-time jobs but does not cover rework jobs.
D. Ace Repairs does not in any way penalize mechanics who have worked on complex repair jobs that later had to be reworked.
E. There is no category of repair jobs in which Ace Repairs invariably carries out first-time jobs satisfactorily.

5. Excavations of the Roman city of Sepphoris have uncovered numerous detailed mosaics depicting several readily identifiable animal species:  a hare, a partridge, and various Mediterranean fish.  Oddly, most of the species represented did not live in the Sepphoris region when these mosaics were created.  Since identical motifs appear in mosaics found in other Roman cities, however, the mosaics of Sepphoris were very likely created by traveling artisans from some other part of the Roman Empire.
Which of the following is an assumption on which the argument depends?
A. The Sepphoris mosaics are not composed exclusively of types of stones found naturally in the Sepphoris area.
B. There is no single region to which all the species depicted in the Sepphoris mosaics are native.
C. No motifs appear in the Sepphoris mosaics that do not also appear in the mosaics of some other Roman city.
D. All of the animal figures in the Sepphoris mosaics are readily identifiable as representations of known species.
E. There was not a common repertory of mosaic designs with which artisans who lived in various parts of the Roman Empire were familiar.

6. Newspaper editorial:
In an attempt to reduce the crime rate, the governor is getting tough on criminals and making prison conditions harsher. Part of this effort has been to deny inmates the access they formerly had to college-level courses.  However, this action is clearly counter to the governor’s ultimate goal, since after being released form prison, inmates who had taken such courses committed far fewer crimes overall than other inmates.
Which of the following is an assumption on which the argument depends?
A.   Not being able to take college-level courses while in prison is unlikely to deter anyone from a crime that he or she might otherwise have committed.
B.   Former inmates are no more likely to commit crimes than are members of the general population.
C.   The group of inmates who chose to take college-level courses were not already less likely than other inmates to commit crimes after being released.
D.   Taking high school level courses in prison has less effect on an inmate’s subsequent behavior than taking college-level courses does.
E.    The governor’s ultimate goal actually is to gain popularity by convincing people that something effective is being done about crime.

7. Radio stations with radio data system (RDS) technology broadcast special program information that only radios with an RDS feature can receive. Between 1994 and 1996, the number of RDS radio stations in Verdland increased from 250 to 600.However, since the number of RDS-equipped radios in Verdland was about the same in 1996 as in 1994, the number of Verdlanders receiving the special program information probably did not increase significantly.
Which of the following is an assumption on which the argument depends?
(A) Few if any of the RDS radio stations that began broadcasting in Verdland after 1994 broadcast to people with RDS-equipped radios living in areas not previously reached by RDS stations.
(B) In 1996 most Verdlanders who lived within the listening area of an RDS station already had a radio equipped to receive RDS.
(C) Equipping a radio station with RDS technology does not decrease the station's listening area.
(D) In 1996 Verdlanders who did not own radios equipped to receive RDS could not receive any programming from the RDS radio stations that began broadcasting in Verdland after 1994.
(E) The RDS radio stations in Verdland in 1996 did not all offer the same type of programming.

8. When people evade income taxes by not declaring taxable income, a vicious cycle results.  Tax evasion forces lawmakers to raise income tax rates, which causes the tax burden on non-evading taxpayers to become heavier.  This, in turn, encourages even more taxpayers to evade income taxes by hiding taxable income.

The vicious cycle described above could not result unless which of the following were true?
(A) An increase in tax rates tends to function as an incentive for taxpayers to try to increase their pretax incomes.
(B) Some methods for detecting tax evaders, and thus recovering some tax revenue lost through evasion, bring in more than they cost, but their success rate varies from year to year.
(C) When lawmakers establish income tax rates in order to generate a certain level of revenue, they do not allow adequately for revenue that will be lost through evasion.
(D) No one who routinely hides some taxable income can be induced by a lowering of tax rates to stop hiding such income unless fines for evaders are raised at the same time.
(E) Taxpayers do not differ from each other with respect to the rate of taxation that will cause them to evade taxes.

9. The average hourly wage of television assemblers in Vernland has long been significantly lower than that in neighboring Borodia. Since Borodia dropped all tariffs on Vernlandian televisions three years ago, the number of televisions sold annually in Borodia has not changed. However, recent statistics show a drop in the number of television assemblers in Borodia. Therefore, updated trade statistics will probably indicate that the number of televisions Borodia imports annually from Vernland has increased.
Which of the following is an assumption on which the argument depends?
(A) The number of television assemblers in Vernland has increased by at least as much as the number of television assemblers in Borodia has decreased.
(B) Televisions assembled in Vernland have features that televisions assembled in Borodia do not have.
(C) The average number of hours it takes a Borodian television assembler to assemble a television has not decreased significantly during the past three years.
(D) The number of televisions assembled annually in Vernland has increased signifi cantly during the past three years.
(E) The difference between the hourly wage of television assemblers in Vernland and the hourly wage of television assemblers in Borodia is likely to decrease in the next few years.


10. Many people suffer an allergic reaction to certain sulfites, including those that are commonly added to wine as preservatives. However, since there are several wine makers who add sulfites to none of the wines they produce, people who would like to drink wine but are allergic to sulfites can drink wines produced by these wine makers without risking an allergic reaction to sulfites.

Which of the following is an assumption on which the argument depends?
A. These wine makers have been able to duplicate the preservative effect produced by adding sulfites by means that do not involve adding any potentially allergenic substances to their wine.
B. Not all forms of sulfite are equally likely to produce the allergic reactions.
C. Wine is the only beverage to which sulfites are commonly added.
D. Apart from sulfites, there are no substances commonly present in wine that give rise to an allergic reaction.
E. Sulfites are not naturally present in the wines produced by these wine makers in amounts large enough to produce an allergic reaction in someone who drinks these wines.
12#
发表于 2011-6-27 09:58:06 | 只看该作者
my answers : B B C A C C D C C E
Waiting for the correct answers ~
13#
 楼主| 发表于 2011-6-27 10:03:58 | 只看该作者
my answers : B B C A C C D C C E
Waiting for the correct answers ~
-- by 会员 jameshzd (2011/6/27 9:58:06)



#2, #5 and #7 need another look.
14#
 楼主| 发表于 2011-6-27 10:04:10 | 只看该作者
1. A recent report determined that although only 3 percent of drivers on Maryland highways equipped their vehicles with radar detectors, 33 percent of all vehicles ticketed for exceeding the speed limit were equipped with them. Clearly, drivers who equip their vehicles with radar detectors are more likely to exceed the speed limit regularly than are drivers who do not.
The conclusion drawn above depends on which of the following assumptions?
(A) Drivers who equip their vehicles with radar detectors are less likely to be ticketed for exceeding the speed limit than are drivers who do not.
(B) Drivers who are ticketed for exceeding the speed limit are more likely to exceed the speed limit regularly than are drivers who are not ticketed.
(C) The number of vehicles that were ticketed for exceeding the speed limit was greater than the number of vehicles that were equipped with radar detectors.
(D) Many of the vehicles that were ticketed for exceeding the speed limit were ticketed more than once in the time period covered by the report.
(E) Drivers on Maryland highways exceeded the speed limit more often than did drivers on other state highways not covered in the report.

If you negate (B), you have:
Drivers who are ticketed for exceeding the speed limit are NOT more likely to exceed the speed limit regularly than are drivers who are not ticketed.
If that is the case, then for those drivers who have radar detectors and who got a speeding ticket, they are NOT more likely to exceed the speed limit regularly than are drivers who are not ticketed. This is in contrary to the conclusion of the stimulus -- Drivers who equip their vehicles with radar detectors ARE more likely to exceed the speed limit regularly than are driver who do not. So if you negate (B), the stimulus falls apart. Thus, (B) is the correct answer.

2. A researcher discovered that people who have low levels of immune-system activity tend to score much lower on tests of mental health than do people with normal or high immune-system activity. The researcher concluded from this experiment that the immune system protects against mental illness as well as against physical disease.
The researcher's conclusion depends on which of the following assumptions?
(A) High immune-system activity protects against mental illness better than normal immune system activity does.
(B) Mental illness is similar to physical disease in its effects on body systems.
(C) People with high immune-system activity cannot develop mental illness.
(D) Mental illness does not cause people's immune system activity to decrease.
(E) Psychological treatment of mental illness is not as effective as is medical treatment.

If you negate (D), you have:
Mental illness DOES cause people's immune system activity to decrease.
If that is true, then the author’s conclusion that the immune system protects against mental illness is wrong. So if you negate (D), the stimulus falls apart. Thus, (D) is the correct answer.

3. In Kravonia, the average salary for jobs requiring a college degree has always been higher than the average salary for jobs that do not require a degree. Current enrollments in Kravonia's colleges indicate that over the next four years the percentage of the Kravonian workforce with college degrees will increase dramatically.  Therefore, the average salary for all workers in Kravonia is likely to increase over the next four years.
Which of the following is an assumption on which the argument depends?
A. Kravonians with more than one college degree earn more, on average, than do Kravonians with only one college degree.
B. The percentage of Kravonians who attend college in order to earn higher salaries is higher now than it was several years ago.
C. The higher average salary for jobs requiring a college degree is not due largely to a scarcity among the Kravonian workforce of people with a college degree.
D. The average salary in Kravonia for jobs that do not require a college degree will not increase over the next four years.
E. Few members of the Kravonian workforce earned their degrees in other countries.

If you negate C), you get:
The higher average salary for jobs requiring a college degree is due largely to a scarcity among the Kravonian workforce of people with a college degree.
If this is the case, with the dramatically increasing percentage of Kravonian workforce with college degrees, the shortage of college graduates will be history, then so will be the higher average salary for jobs requiring a college degree. If this is the case, the conclusion that the average salary for all workers is likely to increase will not hold.

4. A significant number of complex repair jobs carried out by Ace Repairs have to be reworked under the company’s warranty. The reworked jobs are invariably satisfactory. When initial repairs are inadequate, therefore, it is not because the mechanics lack competence; rather, there is clearly a level of focused concentration that complex repairs require that is elicited more reliably by rework jobs than by first-time jobs.
The argument above assumes which of the following?
A. There is no systematic difference in membership between the group of mechanics who do first-time jobs and the group of those who do rework jobs.
B. There is no company that successfully competes with Ace Repairs for complex repair jobs.
C. Ace Repairs’ warranty is good on first-time jobs but does not cover rework jobs.
D. Ace Repairs does not in any way penalize mechanics who have worked on complex repair jobs that later had to be reworked.
E. There is no category of repair jobs in which Ace Repairs invariably carries out first-time jobs satisfactorily.

Part of the main conclusion says: When initial repairs are inadequate, therefore, it is not because the mechanics lack competence.

If you negate A), you have:
There is SOME systematic difference in membership between the group of mechanics who do first-time jobs and the group of those who do rework jobs.
If this is true, there is a difference in skills between the two groups of mechanics and the first group might be less competent than the second and the reason why the first repair fails is due to incompetence. Thus, the conclusion of the argument falls apart.

5. Excavations of the Roman city of Sepphoris have uncovered numerous detailed mosaics depicting several readily identifiable animal species:  a hare, a partridge, and various Mediterranean fish.  Oddly, most of the species represented did not live in the Sepphoris region when these mosaics were created.  Since identical motifs appear in mosaics found in other Roman cities, however, the mosaics of Sepphoris were very likely created by traveling artisans from some other part of the Roman Empire.
Which of the following is an assumption on which the argument depends?
A. The Sepphoris mosaics are not composed exclusively of types of stones found naturally in the Sepphoris area.
B. There is no single region to which all the species depicted in the Sepphoris mosaics are native.
C. No motifs appear in the Sepphoris mosaics that do not also appear in the mosaics of some other Roman city.
D. All of the animal figures in the Sepphoris mosaics are readily identifiable as representations of known species.
E. There was not a common repertory of mosaic designs with which artisans who lived in various parts of the Roman Empire were familiar.

If you negate E), you have:
There was a common repertory of mosaic designs with which artisans who lived in various parts of the Roman Empire were familiar

If so, the artists do not need to travel and would still be able to create the mosaic designs in Seppphoris. Thus the conclusion falls apart.

6. Newspaper editorial:
In an attempt to reduce the crime rate, the governor is getting tough on criminals and making prison conditions harsher. Part of this effort has been to deny inmates the access they formerly had to college-level courses.  However, this action is clearly counter to the governor’s ultimate goal, since after being released form prison, inmates who had taken such courses committed far fewer crimes overall than other inmates.
Which of the following is an assumption on which the argument depends?
A.   Not being able to take college-level courses while in prison is unlikely to deter anyone from a crime that he or she might otherwise have committed.
B.   Former inmates are no more likely to commit crimes than are members of the general population.
C.   The group of inmates who chose to take college-level courses were not already less likely than other inmates to commit crimes after being released.
D.   Taking high school level courses in prison has less effect on an inmate’s subsequent behavior than taking college-level courses does.
E.    The governor’s ultimate goal actually is to gain popularity by convincing people that something effective is being done about crime.

If you negate C), you get: The group of inmates who chose to take college-level courses WERE already less likely than other inmates to commit crimes after being released. If so, these inmates are GOOD inmates to begin with, with or without the college-level course training. Therefore, the argument that allowing an inmate to take such courses in prison would decrease an inmate's chance of committing crimes after his or her release is false.

Thus, C) is the necessary assumption.

7. Radio stations with radio data system (RDS) technology broadcast special program information that only radios with an RDS feature can receive. Between 1994 and 1996, the number of RDS radio stations in Verdland increased from 250 to 600.However, since the number of RDS-equipped radios in Verdland was about the same in 1996 as in 1994, the number of Verdlanders receiving the special program information probably did not increase significantly.
Which of the following is an assumption on which the argument depends?
(A) Few if any of the RDS radio stations that began broadcasting in Verdland after 1994 broadcast to people with RDS-equipped radios living in areas not previously reached by RDS stations.
(B) In 1996 most Verdlanders who lived within the listening area of an RDS station already had a radio equipped to receive RDS.
(C) Equipping a radio station with RDS technology does not decrease the station's listening area.
(D) In 1996 Verdlanders who did not own radios equipped to receive RDS could not receive any programming from the RDS radio stations that began broadcasting in Verdland after 1994.
(E) The RDS radio stations in Verdland in 1996 did not all offer the same type of programming.

If you negate A), you have:
Few if any of the RDS radio stations that began broadcasting in Verdland after 1994 DID NOT broadcast to people with RDS-equipped radios living in areas not previously reached by RDS stations.
This is a double negative statement. If that is true, then almost all these stations broadcast to people with RDS-equipped radios living in areas not previously reached by RDS stations. In other words, stations reached out to NEW listeners! If so, the conclusion “the number of Verdlanders receiving the special program information probably did not increase significantly” is wrong. Thus, A) is the necessary assumption.

8. When people evade income taxes by not declaring taxable income, a vicious cycle results.  Tax evasion forces lawmakers to raise income tax rates, which causes the tax burden on non-evading taxpayers to become heavier.  This, in turn, encourages even more taxpayers to evade income taxes by hiding taxable income.

The vicious cycle described above could not result unless which of the following were true?
(A) An increase in tax rates tends to function as an incentive for taxpayers to try to increase their pretax incomes.
(B) Some methods for detecting tax evaders, and thus recovering some tax revenue lost through evasion, bring in more than they cost, but their success rate varies from year to year.
(C) When lawmakers establish income tax rates in order to generate a certain level of revenue, they do not allow adequately for revenue that will be lost through evasion.
(D) No one who routinely hides some taxable income can be induced by a lowering of tax rates to stop hiding such income unless fines for evaders are raised at the same time.
(E) Taxpayers do not differ from each other with respect to the rate of taxation that will cause them to evade taxes.

If you negate C), you have:
When lawmakers establish income tax rates in order to generate a certain level of revenue, they DO allow adequately for revenue that will be lost through evasion.
If this is true, the vicious cycle of – evading taxes --> tax shortfall --> raising taxes --> more people evading taxes --would not happen, because the trigger of the cycle – evading taxes would lead to tax shortfall – would not occur. Thus, the whole argument falls apart.

9. The average hourly wage of television assemblers in Vernland has long been significantly lower than that in neighboring Borodia. Since Borodia dropped all tariffs on Vernlandian televisions three years ago, the number of televisions sold annually in Borodia has not changed. However, recent statistics show a drop in the number of television assemblers in Borodia. Therefore, updated trade statistics will probably indicate that the number of televisions Borodia imports annually from Vernland has increased.
Which of the following is an assumption on which the argument depends?
(A) The number of television assemblers in Vernland has increased by at least as much as the number of television assemblers in Borodia has decreased.
(B) Televisions assembled in Vernland have features that televisions assembled in Borodia do not have.
(C) The average number of hours it takes a Borodian television assembler to assemble a television has not decreased significantly during the past three years.
(D) The number of televisions assembled annually in Vernland has increased signifi cantly during the past three years.
(E) The difference between the hourly wage of television assemblers in Vernland and the hourly wage of television assemblers in Borodia is likely to decrease in the next few years.

Premise: Televisions are made in both V and B, and B imports TV from V. The number of televisions sold in B keeps constant. These TV's come from both B and V. The number of TV assemblers in B decreases.

Conclusion: TV imports from V to B have increased.

The assumption is that if TV assemblers in B are fewer, there will be fewer TV made in B. A further assumption is that the efficiency of TV assemblers in B has not changed (answer C).

If you use negation method, you would find that after negating C), the argument falls apart. If assemblers in B become more efficient, they can produce the same amount or more TV than before, even with a lower number of total assemblers.

10. Many people suffer an allergic reaction to certain sulfites, including those that are commonly added to wine as preservatives. However, since there are several wine makers who add sulfites to none of the wines they produce, people who would like to drink wine but are allergic to sulfites can drink wines produced by these wine makers without risking an allergic reaction to sulfites.

Which of the following is an assumption on which the argument depends?
A. These wine makers have been able to duplicate the preservative effect produced by adding sulfites by means that do not involve adding any potentially allergenic substances to their wine.
B. Not all forms of sulfite are equally likely to produce the allergic reactions.
C. Wine is the only beverage to which sulfites are commonly added.
D. Apart from sulfites, there are no substances commonly present in wine that give rise to an allergic reaction.
E. Sulfites are not naturally present in the wines produced by these wine makers in amounts large enough to produce an allergic reaction in someone who drinks these wines.

If you negate E), you have:
sulfites ARE naturally present in the wines in large enough amount to cause allergic reaction.
If this is true, those wine without added preservatives are also problematic because the NATURALLY occurring sulfites. Then the argument that allergic people can avoid sulfites by choosing wines without preservatives will not hold. Hence, E) is a necessary assumption.
15#
发表于 2011-6-27 10:48:25 | 只看该作者
Difficult those questions are . Although i know all answers to the questions, i know i am not able to get the right answers when in test. Because i just know the answer, i do not know how to get the right answer and how to solve some types question if all the stuffs in the question are changed. That is, i do not understand the principle of solving those questions.  
For example
question 7
I know the answer is A, but i do not think i can read out the answer in the test day. I even can not figure out what it says unless i read it over and over again. Time is wasted like that.

How can i read out the meaning as like i was reading a Chinese fiction?  how to get ?
16#
 楼主| 发表于 2011-6-27 10:53:11 | 只看该作者
7. Radio stations with radio data system (RDS) technology broadcast special program information that only radios with an RDS feature can receive (background info). Between 1994 and 1996, the number of RDS radio stations in Verdland increased from 250 to 600 (premise 1). However, sincethe number of RDS-equipped radios in Verdland was about the same in 1996 as in 1994 (premise 2), the number of Verdlanders receiving the special program information probably did not increase significantly (main conclusion).

This passage adopts the typical argument structure: Background. Pemise. However, premise, main conclusion. Verdland and Verdlanders are just like Beijing and Beijingness. Train your eyes to spot premises and conclusion. They would help you choose the right answers.

Fore general understanding of English passages, you have to read then re-read difficult passages. No shortcut.
17#
发表于 2011-6-27 11:45:41 | 只看该作者
For question 7

ii state why i choose C: i originally thought that the number of RDS-equipped radios can not dictate the the number of listeners, if new listeners that don't own a RDS-equipped radio can share the RDS-equipped radio with one who owned it . In this way, new listeners can receive the progamming information despite the fixed numeber of  RDS-equipped radios , thus refuting the conclusion.

But after closely examined the option C,  i think it is wrong becasue it cunningly leaves out the word "information" which should have been added after  "could not receive any programming".  As the option stated, if i negate it, it becomes" In 1996 Verdlanders who did not own radios equipped to receive RDS could  receive any programming"  it contradicts the background the argument provided by itself; that is one  who did not own radio is impossible to receive programming because its exclusivity to the RDS-equipped radio . But if c add the word "information" after "could not receive any programming".  Would C  be right ? hope for your suggestion !
18#
发表于 2011-6-27 12:04:47 | 只看该作者
i choose D, not C, i always make such silly mistake.hehe...
19#
 楼主| 发表于 2011-6-27 12:14:53 | 只看该作者
I think D) simply repeats the first sentence of the stimulus, which is background informaion. Therefore, D) is not an assumption, which has to be an unstated premise.
20#
发表于 2011-6-27 12:33:19 | 只看该作者
Thank you Sdcar2010 !  i learn a lot !
您需要登录后才可以回帖 登录 | 立即注册

Mark一下! 看一下! 顶楼主! 感谢分享! 快速回复:

手机版|ChaseDream|GMT+8, 2024-4-20 04:37
京公网安备11010202008513号 京ICP证101109号 京ICP备12012021号

ChaseDream 论坛

© 2003-2023 ChaseDream.com. All Rights Reserved.

返回顶部